Ways to show that $int_{0}^{1}((1-x^r)^{1/r}-x)^ndx=frac{1}{n+1}$ The Next CEO of Stack...

Are police here, aren't itthey?

A small doubt about the dominated convergence theorem

Make solar eclipses exceedingly rare, but still have new moons

How to count occurrences of text in a file?

Would a completely good Muggle be able to use a wand?

Can we say or write : "No, it'sn't"?

Do I need to write [sic] when a number is less than 10 but isn't written out?

Legal workarounds for testamentary trust perceived as unfair

TikZ: How to reverse arrow direction without switching start/end point?

What connection does MS Office have to Netscape Navigator?

I want to delete every two lines after 3rd lines in file contain very large number of lines :

How many extra stops do monopods offer for tele photographs?

Find non-case sensitive string in a mixed list of elements?

Which one is the true statement?

What steps are necessary to read a Modern SSD in Medieval Europe?

Why does standard notation not preserve intervals (visually)

How is this set of matrices closed under multiplication?

Is it ever safe to open a suspicious HTML file (e.g. email attachment)?

Solving system of ODEs with extra parameter

The exact meaning of 'Mom made me a sandwich'

What happened in Rome, when the western empire "fell"?

Is it professional to write unrelated content in an almost-empty email?

Does increasing your ability score affect your main stat?

Newlines in BSD sed vs gsed



Ways to show that $int_{0}^{1}((1-x^r)^{1/r}-x)^ndx=frac{1}{n+1}$



The Next CEO of Stack OverflowHypergeometric identityImaginary part of $int_{0}^{pi/2} frac{x^2}{x^2+log ^2(-2cos x)} :mathrm{d}x$ and $int_{0}^{pi/2} frac{log cos x}{x^2}:mathrm{d}x$A closed form for $int_{0}^{pi/2} x^3 ln^3(2 cos x):mathrm{d}x$Evaluate $int_{0}^{1}(1-x)^ndx$ by expanding the bracket.Show that :$int_{0}^{1}(1-x^2)^ndx={(2n)!!over (2n+1)!!}$Prove that $int_{0}^{1}mathrm{f}left(xright)mathrm{d}x = 1 - gamma$Square of an Integral into Single IntegralOn the integral $int_{0}^{pi/2} frac{x log left ( 1-sin x right )}{sin x} , mathrm{d}x$Closed form of $int_0^infty (frac{arctan(x)}{x})^ndx$Find $p>1$ that ${intlimits^p_1}frac{1}{x},mathrm{d}x={intlimits^p_1}lnleft(xright),mathrm{d}x$Proving that $lim_{ntoinfty}(int_{a}^{b}f(x)^ndx)^{1/n} = max_{xin [a,b]}f(x)$












23












$begingroup$


Through some calculation, I found that for all $r>0$
$$
begin{array}{rcl}
{displaystyleint_{0}^{1}left[left(1 - x^{r}right)^{1/r} - xright]^{2},mathrm{d}x} & {displaystyle =} &
{displaystyle{1 over 3}}
\
{displaystyleint_{0}^{1}left[left(1 - x^{r}right)^{1/r} - xright]^{4},mathrm{d}x} & {displaystyle =} &
{displaystyle{1 over 5}}
\
{displaystyleint_{0}^{1}left[left(1 - x^{r}right)^{1/r} - xright]^{6},mathrm{d}x} & {displaystyle =} &
{displaystyle{1 over 7}}
end{array}
$$



It seems like for $left{n = 2k, r > 0 mid k in mathbb{N}right}$




$$
int_{0}^{1}
left[left(1 - x^{r}right)^{1/r} - xright]^{n}mathrm{d}x = {1 over n + 1}
$$




I want to prove this general form.



Someone suggested to make the substitution
$$y=(1-x^r)^{1/r}$$
So I rewrote the integral into
$$int_{0}^{1}((1-x^r)^{1/r}-x)^ndx=int_{0}^{1}(y-x)^ndx$$
and tried to use the binomial formula:
$$(y-x)^{n}=sum _{k=0}^{n}{binom {n}{k}}(-1)^{n-k}x^{n-k}y^{k}$$



The integral then becomes
$$begin{align}
int_{0}^{1}(y-x)^ndx&=int_{0}^{1}sum _{k=0}^{n}{binom {n}{k}}(-1)^{n-k}x^{n-k}y^{k}dx\
&=sum _{k=0}^{n}{binom {n}{k}}(-1)^{n-k}int_{0}^{1}x^{n-k}y^{k}dx\
&=sum _{k=0}^{n}{binom {n}{k}}(-1)^{n-k}int_{0}^{1}x^{n-k}(1-x^r)^{k/r}dx\
end{align}$$

Now I think I need to use Beta function:
$$B(x,y) = frac{(x-1)!(y-1)!}{(x+y-1)!}= int_{0}^{1}u^{x-1}(1-u)^{y-1}du=sum_{n=0}^{infty}frac{{binom{n-y}{n}}}{x+n}$$
Am I on the right track? Are here any easier ways to prove the general form?










share|cite|improve this question











$endgroup$












  • $begingroup$
    Perhaps the simpler integral (with $r=1$) can be used in an integration-by-parts approach: $frac{Gamma left(1+frac{1}{n}right)^2}{Gamma left(frac{n+2}{n}right)}-frac{1}{2}$
    $endgroup$
    – David G. Stork
    Jan 11 at 23:45










  • $begingroup$
    Note: switching between $n$ and $r$ between the header and the body is very confusing. Sticking with the header notation, and letting, $n=1$ you get the integral $int_0^1 (1-2x)^r,dx$ which is not always $frac 1{r+1}$. If $r=1$, say, you get $0$.
    $endgroup$
    – lulu
    Jan 12 at 0:03










  • $begingroup$
    And, sticking with the header notation and letting $n=2$, we see that $int_0^1 ((1-x^2)^{1/2}-x)^3,dx = frac {3pi}8 -1 approx .1781 neq frac 14$ . Or am I misreading something?
    $endgroup$
    – lulu
    Jan 12 at 0:08






  • 2




    $begingroup$
    I see, thank you for re-asking the question! It is a nice integral for sure.
    $endgroup$
    – Zacky
    Jan 12 at 0:26






  • 1




    $begingroup$
    @clathratus: Oops, $u$ should be $y$. I will edit it. Thanks for pointing that out.
    $endgroup$
    – Larry
    Jan 12 at 0:51


















23












$begingroup$


Through some calculation, I found that for all $r>0$
$$
begin{array}{rcl}
{displaystyleint_{0}^{1}left[left(1 - x^{r}right)^{1/r} - xright]^{2},mathrm{d}x} & {displaystyle =} &
{displaystyle{1 over 3}}
\
{displaystyleint_{0}^{1}left[left(1 - x^{r}right)^{1/r} - xright]^{4},mathrm{d}x} & {displaystyle =} &
{displaystyle{1 over 5}}
\
{displaystyleint_{0}^{1}left[left(1 - x^{r}right)^{1/r} - xright]^{6},mathrm{d}x} & {displaystyle =} &
{displaystyle{1 over 7}}
end{array}
$$



It seems like for $left{n = 2k, r > 0 mid k in mathbb{N}right}$




$$
int_{0}^{1}
left[left(1 - x^{r}right)^{1/r} - xright]^{n}mathrm{d}x = {1 over n + 1}
$$




I want to prove this general form.



Someone suggested to make the substitution
$$y=(1-x^r)^{1/r}$$
So I rewrote the integral into
$$int_{0}^{1}((1-x^r)^{1/r}-x)^ndx=int_{0}^{1}(y-x)^ndx$$
and tried to use the binomial formula:
$$(y-x)^{n}=sum _{k=0}^{n}{binom {n}{k}}(-1)^{n-k}x^{n-k}y^{k}$$



The integral then becomes
$$begin{align}
int_{0}^{1}(y-x)^ndx&=int_{0}^{1}sum _{k=0}^{n}{binom {n}{k}}(-1)^{n-k}x^{n-k}y^{k}dx\
&=sum _{k=0}^{n}{binom {n}{k}}(-1)^{n-k}int_{0}^{1}x^{n-k}y^{k}dx\
&=sum _{k=0}^{n}{binom {n}{k}}(-1)^{n-k}int_{0}^{1}x^{n-k}(1-x^r)^{k/r}dx\
end{align}$$

Now I think I need to use Beta function:
$$B(x,y) = frac{(x-1)!(y-1)!}{(x+y-1)!}= int_{0}^{1}u^{x-1}(1-u)^{y-1}du=sum_{n=0}^{infty}frac{{binom{n-y}{n}}}{x+n}$$
Am I on the right track? Are here any easier ways to prove the general form?










share|cite|improve this question











$endgroup$












  • $begingroup$
    Perhaps the simpler integral (with $r=1$) can be used in an integration-by-parts approach: $frac{Gamma left(1+frac{1}{n}right)^2}{Gamma left(frac{n+2}{n}right)}-frac{1}{2}$
    $endgroup$
    – David G. Stork
    Jan 11 at 23:45










  • $begingroup$
    Note: switching between $n$ and $r$ between the header and the body is very confusing. Sticking with the header notation, and letting, $n=1$ you get the integral $int_0^1 (1-2x)^r,dx$ which is not always $frac 1{r+1}$. If $r=1$, say, you get $0$.
    $endgroup$
    – lulu
    Jan 12 at 0:03










  • $begingroup$
    And, sticking with the header notation and letting $n=2$, we see that $int_0^1 ((1-x^2)^{1/2}-x)^3,dx = frac {3pi}8 -1 approx .1781 neq frac 14$ . Or am I misreading something?
    $endgroup$
    – lulu
    Jan 12 at 0:08






  • 2




    $begingroup$
    I see, thank you for re-asking the question! It is a nice integral for sure.
    $endgroup$
    – Zacky
    Jan 12 at 0:26






  • 1




    $begingroup$
    @clathratus: Oops, $u$ should be $y$. I will edit it. Thanks for pointing that out.
    $endgroup$
    – Larry
    Jan 12 at 0:51
















23












23








23


21



$begingroup$


Through some calculation, I found that for all $r>0$
$$
begin{array}{rcl}
{displaystyleint_{0}^{1}left[left(1 - x^{r}right)^{1/r} - xright]^{2},mathrm{d}x} & {displaystyle =} &
{displaystyle{1 over 3}}
\
{displaystyleint_{0}^{1}left[left(1 - x^{r}right)^{1/r} - xright]^{4},mathrm{d}x} & {displaystyle =} &
{displaystyle{1 over 5}}
\
{displaystyleint_{0}^{1}left[left(1 - x^{r}right)^{1/r} - xright]^{6},mathrm{d}x} & {displaystyle =} &
{displaystyle{1 over 7}}
end{array}
$$



It seems like for $left{n = 2k, r > 0 mid k in mathbb{N}right}$




$$
int_{0}^{1}
left[left(1 - x^{r}right)^{1/r} - xright]^{n}mathrm{d}x = {1 over n + 1}
$$




I want to prove this general form.



Someone suggested to make the substitution
$$y=(1-x^r)^{1/r}$$
So I rewrote the integral into
$$int_{0}^{1}((1-x^r)^{1/r}-x)^ndx=int_{0}^{1}(y-x)^ndx$$
and tried to use the binomial formula:
$$(y-x)^{n}=sum _{k=0}^{n}{binom {n}{k}}(-1)^{n-k}x^{n-k}y^{k}$$



The integral then becomes
$$begin{align}
int_{0}^{1}(y-x)^ndx&=int_{0}^{1}sum _{k=0}^{n}{binom {n}{k}}(-1)^{n-k}x^{n-k}y^{k}dx\
&=sum _{k=0}^{n}{binom {n}{k}}(-1)^{n-k}int_{0}^{1}x^{n-k}y^{k}dx\
&=sum _{k=0}^{n}{binom {n}{k}}(-1)^{n-k}int_{0}^{1}x^{n-k}(1-x^r)^{k/r}dx\
end{align}$$

Now I think I need to use Beta function:
$$B(x,y) = frac{(x-1)!(y-1)!}{(x+y-1)!}= int_{0}^{1}u^{x-1}(1-u)^{y-1}du=sum_{n=0}^{infty}frac{{binom{n-y}{n}}}{x+n}$$
Am I on the right track? Are here any easier ways to prove the general form?










share|cite|improve this question











$endgroup$




Through some calculation, I found that for all $r>0$
$$
begin{array}{rcl}
{displaystyleint_{0}^{1}left[left(1 - x^{r}right)^{1/r} - xright]^{2},mathrm{d}x} & {displaystyle =} &
{displaystyle{1 over 3}}
\
{displaystyleint_{0}^{1}left[left(1 - x^{r}right)^{1/r} - xright]^{4},mathrm{d}x} & {displaystyle =} &
{displaystyle{1 over 5}}
\
{displaystyleint_{0}^{1}left[left(1 - x^{r}right)^{1/r} - xright]^{6},mathrm{d}x} & {displaystyle =} &
{displaystyle{1 over 7}}
end{array}
$$



It seems like for $left{n = 2k, r > 0 mid k in mathbb{N}right}$




$$
int_{0}^{1}
left[left(1 - x^{r}right)^{1/r} - xright]^{n}mathrm{d}x = {1 over n + 1}
$$




I want to prove this general form.



Someone suggested to make the substitution
$$y=(1-x^r)^{1/r}$$
So I rewrote the integral into
$$int_{0}^{1}((1-x^r)^{1/r}-x)^ndx=int_{0}^{1}(y-x)^ndx$$
and tried to use the binomial formula:
$$(y-x)^{n}=sum _{k=0}^{n}{binom {n}{k}}(-1)^{n-k}x^{n-k}y^{k}$$



The integral then becomes
$$begin{align}
int_{0}^{1}(y-x)^ndx&=int_{0}^{1}sum _{k=0}^{n}{binom {n}{k}}(-1)^{n-k}x^{n-k}y^{k}dx\
&=sum _{k=0}^{n}{binom {n}{k}}(-1)^{n-k}int_{0}^{1}x^{n-k}y^{k}dx\
&=sum _{k=0}^{n}{binom {n}{k}}(-1)^{n-k}int_{0}^{1}x^{n-k}(1-x^r)^{k/r}dx\
end{align}$$

Now I think I need to use Beta function:
$$B(x,y) = frac{(x-1)!(y-1)!}{(x+y-1)!}= int_{0}^{1}u^{x-1}(1-u)^{y-1}du=sum_{n=0}^{infty}frac{{binom{n-y}{n}}}{x+n}$$
Am I on the right track? Are here any easier ways to prove the general form?







calculus integration definite-integrals






share|cite|improve this question















share|cite|improve this question













share|cite|improve this question




share|cite|improve this question








edited Mar 5 at 0:48









Felix Marin

68.9k7109146




68.9k7109146










asked Jan 11 at 23:26









LarryLarry

2,53031131




2,53031131












  • $begingroup$
    Perhaps the simpler integral (with $r=1$) can be used in an integration-by-parts approach: $frac{Gamma left(1+frac{1}{n}right)^2}{Gamma left(frac{n+2}{n}right)}-frac{1}{2}$
    $endgroup$
    – David G. Stork
    Jan 11 at 23:45










  • $begingroup$
    Note: switching between $n$ and $r$ between the header and the body is very confusing. Sticking with the header notation, and letting, $n=1$ you get the integral $int_0^1 (1-2x)^r,dx$ which is not always $frac 1{r+1}$. If $r=1$, say, you get $0$.
    $endgroup$
    – lulu
    Jan 12 at 0:03










  • $begingroup$
    And, sticking with the header notation and letting $n=2$, we see that $int_0^1 ((1-x^2)^{1/2}-x)^3,dx = frac {3pi}8 -1 approx .1781 neq frac 14$ . Or am I misreading something?
    $endgroup$
    – lulu
    Jan 12 at 0:08






  • 2




    $begingroup$
    I see, thank you for re-asking the question! It is a nice integral for sure.
    $endgroup$
    – Zacky
    Jan 12 at 0:26






  • 1




    $begingroup$
    @clathratus: Oops, $u$ should be $y$. I will edit it. Thanks for pointing that out.
    $endgroup$
    – Larry
    Jan 12 at 0:51




















  • $begingroup$
    Perhaps the simpler integral (with $r=1$) can be used in an integration-by-parts approach: $frac{Gamma left(1+frac{1}{n}right)^2}{Gamma left(frac{n+2}{n}right)}-frac{1}{2}$
    $endgroup$
    – David G. Stork
    Jan 11 at 23:45










  • $begingroup$
    Note: switching between $n$ and $r$ between the header and the body is very confusing. Sticking with the header notation, and letting, $n=1$ you get the integral $int_0^1 (1-2x)^r,dx$ which is not always $frac 1{r+1}$. If $r=1$, say, you get $0$.
    $endgroup$
    – lulu
    Jan 12 at 0:03










  • $begingroup$
    And, sticking with the header notation and letting $n=2$, we see that $int_0^1 ((1-x^2)^{1/2}-x)^3,dx = frac {3pi}8 -1 approx .1781 neq frac 14$ . Or am I misreading something?
    $endgroup$
    – lulu
    Jan 12 at 0:08






  • 2




    $begingroup$
    I see, thank you for re-asking the question! It is a nice integral for sure.
    $endgroup$
    – Zacky
    Jan 12 at 0:26






  • 1




    $begingroup$
    @clathratus: Oops, $u$ should be $y$. I will edit it. Thanks for pointing that out.
    $endgroup$
    – Larry
    Jan 12 at 0:51


















$begingroup$
Perhaps the simpler integral (with $r=1$) can be used in an integration-by-parts approach: $frac{Gamma left(1+frac{1}{n}right)^2}{Gamma left(frac{n+2}{n}right)}-frac{1}{2}$
$endgroup$
– David G. Stork
Jan 11 at 23:45




$begingroup$
Perhaps the simpler integral (with $r=1$) can be used in an integration-by-parts approach: $frac{Gamma left(1+frac{1}{n}right)^2}{Gamma left(frac{n+2}{n}right)}-frac{1}{2}$
$endgroup$
– David G. Stork
Jan 11 at 23:45












$begingroup$
Note: switching between $n$ and $r$ between the header and the body is very confusing. Sticking with the header notation, and letting, $n=1$ you get the integral $int_0^1 (1-2x)^r,dx$ which is not always $frac 1{r+1}$. If $r=1$, say, you get $0$.
$endgroup$
– lulu
Jan 12 at 0:03




$begingroup$
Note: switching between $n$ and $r$ between the header and the body is very confusing. Sticking with the header notation, and letting, $n=1$ you get the integral $int_0^1 (1-2x)^r,dx$ which is not always $frac 1{r+1}$. If $r=1$, say, you get $0$.
$endgroup$
– lulu
Jan 12 at 0:03












$begingroup$
And, sticking with the header notation and letting $n=2$, we see that $int_0^1 ((1-x^2)^{1/2}-x)^3,dx = frac {3pi}8 -1 approx .1781 neq frac 14$ . Or am I misreading something?
$endgroup$
– lulu
Jan 12 at 0:08




$begingroup$
And, sticking with the header notation and letting $n=2$, we see that $int_0^1 ((1-x^2)^{1/2}-x)^3,dx = frac {3pi}8 -1 approx .1781 neq frac 14$ . Or am I misreading something?
$endgroup$
– lulu
Jan 12 at 0:08




2




2




$begingroup$
I see, thank you for re-asking the question! It is a nice integral for sure.
$endgroup$
– Zacky
Jan 12 at 0:26




$begingroup$
I see, thank you for re-asking the question! It is a nice integral for sure.
$endgroup$
– Zacky
Jan 12 at 0:26




1




1




$begingroup$
@clathratus: Oops, $u$ should be $y$. I will edit it. Thanks for pointing that out.
$endgroup$
– Larry
Jan 12 at 0:51






$begingroup$
@clathratus: Oops, $u$ should be $y$. I will edit it. Thanks for pointing that out.
$endgroup$
– Larry
Jan 12 at 0:51












4 Answers
4






active

oldest

votes


















35





+50







$begingroup$

We present 3 different solutions.





Solution 1 - slick substitution. We prove a more general statement:




Proposition. Let $R in (0, infty]$ and let $varphi : [0, R] to [0, R]$ satisfy the following conditions:





  • $varphi$ is continuous on $[0, R]$;


  • $varphi(0) = R$ and $varphi(R) = 0$;


  • $varphi$ is bijective and $varphi^{-1} = varphi$.


Then for any integrable function $f$ on $[0, R]$,
$$ int_{0}^{R} f(|x-varphi(x)|) , mathrm{d}x = int_{0}^{R} f(x) , mathrm{d}x. $$




Proof. In case $varphi$ is also continuously differentiable on $(0, R)$, by the substitution $y = varphi(x)$, or equivalently, $x = varphi(y)$,



$$ I
:= int_{0}^{R} f( |x - varphi(x)| ) , mathrm{d}x
= -int_{0}^{R} f( |varphi(y) - y| ) varphi'(y) , mathrm{d}y. $$



Summing two integrals,



begin{align*}
2I
&= int_{0}^{R} f( |x - varphi(x)| ) (1 - varphi'(x)) , mathrm{d}x \
&= int_{-R}^{R} f( |u| ) , mathrm{d}u = 2int_{0}^{R} f(u) , mathrm{d}u, tag{$u = x - varphi(x)$}
end{align*}



proving the claim when $varphi$ is continuously differentiable. This proof can be easily adapted to general $varphi$ by using Stieltjes integral. ■



Now plug $varphi(x) = (1-x^r)^{1/r}$ with $R = 1$ and $f(x) = x^n$ for positive even integer $n$. Then



$$ int_{0}^{1} left( (1-x^r)^{1/r} - x right)^n , mathrm{d}x
= int_{0}^{1} left| x - (1-x^r)^{1/r} right|^n , mathrm{d}x
= int_{0}^{1} x^n , mathrm{d}x
= frac{1}{n+1}. $$





Solution 2 - using beta function. Here is an alternative solution. Write $p = 1/r$. Then using the substitution $x = u^p$,



begin{align*}
int_{0}^{1} left( (1 - x^r)^{1/r} - x right)^n , mathrm{d}x
&= int_{0}^{1} left( (1 - u)^{p} - u^p right)^{n} pu^{p-1} , mathrm{d}u \
&= sum_{k=0}^{n} (-1)^k binom{n}{k} p int_{0}^{1} (1-u)^{p(n-k)} u^{p(k+1)-1} , mathrm{d}u \
&= sum_{k=0}^{n} (-1)^k binom{n}{k} p cdot frac{(p(n-k))!(p(k+1)-1)!}{(p(n+1))!}
end{align*}



Here, $s! = Gamma(s+1)$. Now define $a_k = (pk)!/k!$. Then the above sum simplifies to



begin{align*}
int_{0}^{1} left( (1 - x^r)^{1/r} - x right)^n , mathrm{d}x
&= frac{1}{(n+1)a_{n+1}} sum_{k=0}^{n} (-1)^k a_{n-k}a_{k+1} \
&= frac{1}{(n+1)a_{n+1}} left( a_0 a_{n+1} + sum_{k=0}^{n-1} (-1)^k a_{n-k}a_{k+1} right).
end{align*}



So it suffices to show that $sum_{k=0}^{n-1} (-1)^k a_{n-k}a_{k+1} = 0$. But by the substitution $l = n-1-k$, we have



$$ sum_{k=0}^{n-1} (-1)^k a_{n-k}a_{k+1}
= - sum_{l=0}^{n-1} (-1)^l a_{l+1}a_{n-l}. $$



(Here the parity of $n$ is used.) So the sum equals its negation, hence is zero as required.





Solution 3 - using multivariate calculus. Let $mathcal{C}_r$ denote the curve defined by $x^r + y^r = 1$ in the first quadrant, oriented to the right. Then



$$ I(r) := int_{0}^{1} left( (1 -x^r)^{1/r} - x right)^n , mathrm{d}x
= int_{mathcal{C}_r} ( y - x )^n , mathrm{d}x. $$



Notice that if $0 < r < s$, then $mathcal{C}_s$ lies above $mathcal{C}_r$, and so, the curve $mathcal{C}_r - mathcal{C}_s$ bounds some region, which we denote by $mathcal{D}$, counter-clockwise:



$hspace{10em}$ Region D and two enclosing curves



Then by Green's theorem,



$$ I(r) - I(s)
= int_{partial mathcal{D}} ( y - x )^n , mathrm{d}x
= - iint_{mathcal{D}} n (y - x)^{n-1} , mathrm{d}xmathrm{d}y. $$



But since the region $mathcal{D}$ is symmetric around $y = x$ and $n$ is even, interchanging the roles of $x$ and $y$ shows



$$ iint_{mathcal{D}} n (y - x)^{n-1} , mathrm{d}xmathrm{d}y
= iint_{mathcal{D}} n (x - y)^{n-1} , mathrm{d}xmathrm{d}y
= - iint_{mathcal{D}} n (y - x)^{n-1} , mathrm{d}xmathrm{d}y. $$



Therefore $I(r) = I(s)$ for any $ r < s$, and in particular, letting $s to infty$ gives



$$ I(r) = int _{0}^{1} (1 - x)^n , mathrm{d}x = frac{1}{n+1}. $$






share|cite|improve this answer











$endgroup$









  • 2




    $begingroup$
    Beautiful! This looks like Glasser's Master theorem little brother :D
    $endgroup$
    – Zacky
    Jan 12 at 1:02












  • $begingroup$
    Was it necessary to start with absolute value in the argument of function?
    $endgroup$
    – user
    Jan 12 at 1:04










  • $begingroup$
    @user, It is kind of necessary, in the sense that $$ int_{0}^{1} gleft(x-(1-x^r)^{1/r}right) , mathrm{d}x = int_{0}^{1} g(u) , mathrm{d}u $$ may fail if $g$ is not an even function on $[-1, 1]$.
    $endgroup$
    – Sangchul Lee
    Jan 12 at 1:06








  • 1




    $begingroup$
    @Zacky, Both Glasser's master theorem and my answer deals with specific examples of measure-preserving transformations, hence the conclusion should look similar. Of course, the beauty of Glasser's result is that its proof is very elementary. (The result itself was known much prior to his paper.)
    $endgroup$
    – Sangchul Lee
    Jan 12 at 1:12








  • 2




    $begingroup$
    @ersh, Thank you, and don't feel ashamed :) I was just lucky enough to find these approaches. It is like seeing facebook postings, which are basically a collage of someone else's finest moments...
    $endgroup$
    – Sangchul Lee
    Mar 17 at 7:37



















4












$begingroup$

Here's your Beta integral
$$S=int_0^1x^{n-k}(1-x^r)^{k/r}dx$$
Setting $w=x^r$, we see that
$$S=frac1rint_0^1w^{frac{n+1-k-r}r}(1-w)^{k/r}dw$$
$$S=frac1rint_0^1w^{frac{n+1-k}r-1}(1-w)^{frac{k+r}r-1}dw$$
$$S=frac1rmathrm{B}bigg(frac{n+1-k}r,frac{k+r}rbigg)$$
So
$$I(r,n)=int_0^1[(1-x^r)^{1/r}-x]^ndx$$
$$I(r,n)=frac1rsum_{k=0}^{n}(-1)^{n-k}{nchoose k}frac{Gamma(frac{n+1-k}r)Gamma(frac{k+r}r)}{Gamma(1+frac{n+1}r)}$$
$$I(r,n)=frac1{r}frac{Gamma(n+1)}{Gamma(1+frac{n+1}r)}sum_{k=0}^{n}(-1)^{n-k}frac{Gamma(frac{n+1-k}r)Gamma(frac{k+r}r)}{Gamma(k+1)Gamma(n-k+1)}$$



Which is a closed form






share|cite|improve this answer











$endgroup$









  • 1




    $begingroup$
    Try to find an error in your derivation as $I(r,n)=frac{1}{n+1}$ for even $n$ and any $r$.
    $endgroup$
    – user
    Jan 12 at 1:45












  • $begingroup$
    @user Ah yes I forgot $n$ was even
    $endgroup$
    – clathratus
    Jan 12 at 1:57






  • 1




    $begingroup$
    Nice approach. Seems almost obvious after reading it, but I was stuck when I first saw it (+1)
    $endgroup$
    – user150203
    Jan 12 at 5:31



















4












$begingroup$

Here's a proof with Hypergeometirc function.



We have
$$
underset{j=1}{overset{2 n+1}{sum }}
left(
begin{array}{c}
2 n \
j-1 \
end{array}
right)
(-x)^{j-1}
left(left(1-x^rright)^{1/r}right)^{-j+2 n+1}
=left(left(1-x^rright)^{1/r}-xright)^{2 n}
$$

by binomial expansion.



It is easy to verify that
$$
left(
begin{array}{c}
2 n \
j-1 \
end{array}
right)
(-x)^{j-1}
left(left(1-x^rright)^{1/r}right)^{-j+2 n+1}
=
frac{mathrm d}{mathrm d x}left(
frac{1}{2 n+1}
(-1)^{j+1} x^j binom{2 n+1}{j} , _2F_1left(frac{j}{r},-frac{-j+2 n+1}{r};frac{j}{r}+1;x^rright)
right)
$$

Therefore, we have
$$
int((1-x^r)^{1/r}-x)^{2 n} mathrm dx
=
sum _{j=1}^{2 n+1} frac{1}{2 n+1} (-1)^{j+1} x^j binom{2 n+1}{j} , _2F_1left(frac{j}{r},-frac{-j+2 n+1}{r};frac{j}{r}+1;x^rright).
$$

When $j=2n+1$, the summand in the right hand equals $frac{x^{2 n+1}}{2 n+1}$. This is the term which gives us $frac 1 {2n+1}$.






share|cite|improve this answer









$endgroup$





















    3












    $begingroup$

    $newcommand{bbx}[1]{,bbox[15px,border:1px groove navy]{displaystyle{#1}},}
    newcommand{braces}[1]{leftlbrace,{#1},rightrbrace}
    newcommand{bracks}[1]{leftlbrack,{#1},rightrbrack}
    newcommand{dd}{mathrm{d}}
    newcommand{ds}[1]{displaystyle{#1}}
    newcommand{expo}[1]{,mathrm{e}^{#1},}
    newcommand{ic}{mathrm{i}}
    newcommand{mc}[1]{mathcal{#1}}
    newcommand{mrm}[1]{mathrm{#1}}
    newcommand{pars}[1]{left(,{#1},right)}
    newcommand{partiald}[3][]{frac{partial^{#1} #2}{partial #3^{#1}}}
    newcommand{root}[2][]{,sqrt[#1]{,{#2},},}
    newcommand{totald}[3][]{frac{mathrm{d}^{#1} #2}{mathrm{d} #3^{#1}}}
    newcommand{verts}[1]{leftvert,{#1},rightvert}$

    begin{align}
    &bbox[10px,#ffd]{left.int_{0}^{1}bracks{pars{1 - x^{r}}^{1/r} - x}^{2k},dd x,rightvert_{{large r > 0} atop
    {large k in mathbb{N}_{geq 0}}}}
    \[5mm] stackrel{x^{large r} mapsto x}{=},,,&
    int_{0}^{1}bracks{pars{1 - x}^{1/r} - x^{1/r}}^{2k},{1 over r},
    x^{1/r - 1},dd x
    \[5mm]
    ,,,stackrel{x mapsto x + 1/2}{=},,,&
    {1 over r}int_{-1/2}^{1/2}bracks{pars{{1 over 2} - x}^{1/r} - pars{{1 over 2} + x}^{1/r}}^{2k},
    pars{{1 over 2} + x}^{1/r - 1},dd x
    \[8mm] = &
    {1 over r}int_{0}^{1/2}bracks{pars{{1 over 2} - x}^{1/r} - pars{{1 over 2} + x!}^{1/r}}^{2k}times
    \[2mm] &
    phantom{{1 over r}int_{0}^{1/2}}bracks{pars{{1 over 2} + x}^{1/r - 1} +
    pars{{1 over 2} - x}^{1/r - 1}!}!dd x
    \[8mm] = &
    -int_{0}^{1/2}{1 over 2k + 1},partiald{}{x}bracks{pars{{1 over 2} - x}^{1/r} - pars{{1 over 2} + x}^{1/r}}^{2k + 1},dd x
    \[5mm] = &
    underbrace{braces{-bracks{pars{{1 over 2} - x}^{1/r} - pars{{1 over 2} + x}^{1/r}}^{2k + 1}}_{x = 0}^{x = 1/2}}
    _{ds{= 1 - 0 = 1}},,,{1 over 2k + 1}
    \[5mm] = &
    bbx{1 over 2k + 1}
    end{align}






    share|cite|improve this answer











    $endgroup$













    • $begingroup$
      Nice solution, (+1).
      $endgroup$
      – Larry
      Mar 5 at 13:19










    • $begingroup$
      Thanks @Larry .
      $endgroup$
      – Felix Marin
      Mar 5 at 15:23












    Your Answer





    StackExchange.ifUsing("editor", function () {
    return StackExchange.using("mathjaxEditing", function () {
    StackExchange.MarkdownEditor.creationCallbacks.add(function (editor, postfix) {
    StackExchange.mathjaxEditing.prepareWmdForMathJax(editor, postfix, [["$", "$"], ["\\(","\\)"]]);
    });
    });
    }, "mathjax-editing");

    StackExchange.ready(function() {
    var channelOptions = {
    tags: "".split(" "),
    id: "69"
    };
    initTagRenderer("".split(" "), "".split(" "), channelOptions);

    StackExchange.using("externalEditor", function() {
    // Have to fire editor after snippets, if snippets enabled
    if (StackExchange.settings.snippets.snippetsEnabled) {
    StackExchange.using("snippets", function() {
    createEditor();
    });
    }
    else {
    createEditor();
    }
    });

    function createEditor() {
    StackExchange.prepareEditor({
    heartbeatType: 'answer',
    autoActivateHeartbeat: false,
    convertImagesToLinks: true,
    noModals: true,
    showLowRepImageUploadWarning: true,
    reputationToPostImages: 10,
    bindNavPrevention: true,
    postfix: "",
    imageUploader: {
    brandingHtml: "Powered by u003ca class="icon-imgur-white" href="https://imgur.com/"u003eu003c/au003e",
    contentPolicyHtml: "User contributions licensed under u003ca href="https://creativecommons.org/licenses/by-sa/3.0/"u003ecc by-sa 3.0 with attribution requiredu003c/au003e u003ca href="https://stackoverflow.com/legal/content-policy"u003e(content policy)u003c/au003e",
    allowUrls: true
    },
    noCode: true, onDemand: true,
    discardSelector: ".discard-answer"
    ,immediatelyShowMarkdownHelp:true
    });


    }
    });














    draft saved

    draft discarded


















    StackExchange.ready(
    function () {
    StackExchange.openid.initPostLogin('.new-post-login', 'https%3a%2f%2fmath.stackexchange.com%2fquestions%2f3070440%2fways-to-show-that-int-011-xr1-r-xndx-frac1n1%23new-answer', 'question_page');
    }
    );

    Post as a guest















    Required, but never shown

























    4 Answers
    4






    active

    oldest

    votes








    4 Answers
    4






    active

    oldest

    votes









    active

    oldest

    votes






    active

    oldest

    votes









    35





    +50







    $begingroup$

    We present 3 different solutions.





    Solution 1 - slick substitution. We prove a more general statement:




    Proposition. Let $R in (0, infty]$ and let $varphi : [0, R] to [0, R]$ satisfy the following conditions:





    • $varphi$ is continuous on $[0, R]$;


    • $varphi(0) = R$ and $varphi(R) = 0$;


    • $varphi$ is bijective and $varphi^{-1} = varphi$.


    Then for any integrable function $f$ on $[0, R]$,
    $$ int_{0}^{R} f(|x-varphi(x)|) , mathrm{d}x = int_{0}^{R} f(x) , mathrm{d}x. $$




    Proof. In case $varphi$ is also continuously differentiable on $(0, R)$, by the substitution $y = varphi(x)$, or equivalently, $x = varphi(y)$,



    $$ I
    := int_{0}^{R} f( |x - varphi(x)| ) , mathrm{d}x
    = -int_{0}^{R} f( |varphi(y) - y| ) varphi'(y) , mathrm{d}y. $$



    Summing two integrals,



    begin{align*}
    2I
    &= int_{0}^{R} f( |x - varphi(x)| ) (1 - varphi'(x)) , mathrm{d}x \
    &= int_{-R}^{R} f( |u| ) , mathrm{d}u = 2int_{0}^{R} f(u) , mathrm{d}u, tag{$u = x - varphi(x)$}
    end{align*}



    proving the claim when $varphi$ is continuously differentiable. This proof can be easily adapted to general $varphi$ by using Stieltjes integral. ■



    Now plug $varphi(x) = (1-x^r)^{1/r}$ with $R = 1$ and $f(x) = x^n$ for positive even integer $n$. Then



    $$ int_{0}^{1} left( (1-x^r)^{1/r} - x right)^n , mathrm{d}x
    = int_{0}^{1} left| x - (1-x^r)^{1/r} right|^n , mathrm{d}x
    = int_{0}^{1} x^n , mathrm{d}x
    = frac{1}{n+1}. $$





    Solution 2 - using beta function. Here is an alternative solution. Write $p = 1/r$. Then using the substitution $x = u^p$,



    begin{align*}
    int_{0}^{1} left( (1 - x^r)^{1/r} - x right)^n , mathrm{d}x
    &= int_{0}^{1} left( (1 - u)^{p} - u^p right)^{n} pu^{p-1} , mathrm{d}u \
    &= sum_{k=0}^{n} (-1)^k binom{n}{k} p int_{0}^{1} (1-u)^{p(n-k)} u^{p(k+1)-1} , mathrm{d}u \
    &= sum_{k=0}^{n} (-1)^k binom{n}{k} p cdot frac{(p(n-k))!(p(k+1)-1)!}{(p(n+1))!}
    end{align*}



    Here, $s! = Gamma(s+1)$. Now define $a_k = (pk)!/k!$. Then the above sum simplifies to



    begin{align*}
    int_{0}^{1} left( (1 - x^r)^{1/r} - x right)^n , mathrm{d}x
    &= frac{1}{(n+1)a_{n+1}} sum_{k=0}^{n} (-1)^k a_{n-k}a_{k+1} \
    &= frac{1}{(n+1)a_{n+1}} left( a_0 a_{n+1} + sum_{k=0}^{n-1} (-1)^k a_{n-k}a_{k+1} right).
    end{align*}



    So it suffices to show that $sum_{k=0}^{n-1} (-1)^k a_{n-k}a_{k+1} = 0$. But by the substitution $l = n-1-k$, we have



    $$ sum_{k=0}^{n-1} (-1)^k a_{n-k}a_{k+1}
    = - sum_{l=0}^{n-1} (-1)^l a_{l+1}a_{n-l}. $$



    (Here the parity of $n$ is used.) So the sum equals its negation, hence is zero as required.





    Solution 3 - using multivariate calculus. Let $mathcal{C}_r$ denote the curve defined by $x^r + y^r = 1$ in the first quadrant, oriented to the right. Then



    $$ I(r) := int_{0}^{1} left( (1 -x^r)^{1/r} - x right)^n , mathrm{d}x
    = int_{mathcal{C}_r} ( y - x )^n , mathrm{d}x. $$



    Notice that if $0 < r < s$, then $mathcal{C}_s$ lies above $mathcal{C}_r$, and so, the curve $mathcal{C}_r - mathcal{C}_s$ bounds some region, which we denote by $mathcal{D}$, counter-clockwise:



    $hspace{10em}$ Region D and two enclosing curves



    Then by Green's theorem,



    $$ I(r) - I(s)
    = int_{partial mathcal{D}} ( y - x )^n , mathrm{d}x
    = - iint_{mathcal{D}} n (y - x)^{n-1} , mathrm{d}xmathrm{d}y. $$



    But since the region $mathcal{D}$ is symmetric around $y = x$ and $n$ is even, interchanging the roles of $x$ and $y$ shows



    $$ iint_{mathcal{D}} n (y - x)^{n-1} , mathrm{d}xmathrm{d}y
    = iint_{mathcal{D}} n (x - y)^{n-1} , mathrm{d}xmathrm{d}y
    = - iint_{mathcal{D}} n (y - x)^{n-1} , mathrm{d}xmathrm{d}y. $$



    Therefore $I(r) = I(s)$ for any $ r < s$, and in particular, letting $s to infty$ gives



    $$ I(r) = int _{0}^{1} (1 - x)^n , mathrm{d}x = frac{1}{n+1}. $$






    share|cite|improve this answer











    $endgroup$









    • 2




      $begingroup$
      Beautiful! This looks like Glasser's Master theorem little brother :D
      $endgroup$
      – Zacky
      Jan 12 at 1:02












    • $begingroup$
      Was it necessary to start with absolute value in the argument of function?
      $endgroup$
      – user
      Jan 12 at 1:04










    • $begingroup$
      @user, It is kind of necessary, in the sense that $$ int_{0}^{1} gleft(x-(1-x^r)^{1/r}right) , mathrm{d}x = int_{0}^{1} g(u) , mathrm{d}u $$ may fail if $g$ is not an even function on $[-1, 1]$.
      $endgroup$
      – Sangchul Lee
      Jan 12 at 1:06








    • 1




      $begingroup$
      @Zacky, Both Glasser's master theorem and my answer deals with specific examples of measure-preserving transformations, hence the conclusion should look similar. Of course, the beauty of Glasser's result is that its proof is very elementary. (The result itself was known much prior to his paper.)
      $endgroup$
      – Sangchul Lee
      Jan 12 at 1:12








    • 2




      $begingroup$
      @ersh, Thank you, and don't feel ashamed :) I was just lucky enough to find these approaches. It is like seeing facebook postings, which are basically a collage of someone else's finest moments...
      $endgroup$
      – Sangchul Lee
      Mar 17 at 7:37
















    35





    +50







    $begingroup$

    We present 3 different solutions.





    Solution 1 - slick substitution. We prove a more general statement:




    Proposition. Let $R in (0, infty]$ and let $varphi : [0, R] to [0, R]$ satisfy the following conditions:





    • $varphi$ is continuous on $[0, R]$;


    • $varphi(0) = R$ and $varphi(R) = 0$;


    • $varphi$ is bijective and $varphi^{-1} = varphi$.


    Then for any integrable function $f$ on $[0, R]$,
    $$ int_{0}^{R} f(|x-varphi(x)|) , mathrm{d}x = int_{0}^{R} f(x) , mathrm{d}x. $$




    Proof. In case $varphi$ is also continuously differentiable on $(0, R)$, by the substitution $y = varphi(x)$, or equivalently, $x = varphi(y)$,



    $$ I
    := int_{0}^{R} f( |x - varphi(x)| ) , mathrm{d}x
    = -int_{0}^{R} f( |varphi(y) - y| ) varphi'(y) , mathrm{d}y. $$



    Summing two integrals,



    begin{align*}
    2I
    &= int_{0}^{R} f( |x - varphi(x)| ) (1 - varphi'(x)) , mathrm{d}x \
    &= int_{-R}^{R} f( |u| ) , mathrm{d}u = 2int_{0}^{R} f(u) , mathrm{d}u, tag{$u = x - varphi(x)$}
    end{align*}



    proving the claim when $varphi$ is continuously differentiable. This proof can be easily adapted to general $varphi$ by using Stieltjes integral. ■



    Now plug $varphi(x) = (1-x^r)^{1/r}$ with $R = 1$ and $f(x) = x^n$ for positive even integer $n$. Then



    $$ int_{0}^{1} left( (1-x^r)^{1/r} - x right)^n , mathrm{d}x
    = int_{0}^{1} left| x - (1-x^r)^{1/r} right|^n , mathrm{d}x
    = int_{0}^{1} x^n , mathrm{d}x
    = frac{1}{n+1}. $$





    Solution 2 - using beta function. Here is an alternative solution. Write $p = 1/r$. Then using the substitution $x = u^p$,



    begin{align*}
    int_{0}^{1} left( (1 - x^r)^{1/r} - x right)^n , mathrm{d}x
    &= int_{0}^{1} left( (1 - u)^{p} - u^p right)^{n} pu^{p-1} , mathrm{d}u \
    &= sum_{k=0}^{n} (-1)^k binom{n}{k} p int_{0}^{1} (1-u)^{p(n-k)} u^{p(k+1)-1} , mathrm{d}u \
    &= sum_{k=0}^{n} (-1)^k binom{n}{k} p cdot frac{(p(n-k))!(p(k+1)-1)!}{(p(n+1))!}
    end{align*}



    Here, $s! = Gamma(s+1)$. Now define $a_k = (pk)!/k!$. Then the above sum simplifies to



    begin{align*}
    int_{0}^{1} left( (1 - x^r)^{1/r} - x right)^n , mathrm{d}x
    &= frac{1}{(n+1)a_{n+1}} sum_{k=0}^{n} (-1)^k a_{n-k}a_{k+1} \
    &= frac{1}{(n+1)a_{n+1}} left( a_0 a_{n+1} + sum_{k=0}^{n-1} (-1)^k a_{n-k}a_{k+1} right).
    end{align*}



    So it suffices to show that $sum_{k=0}^{n-1} (-1)^k a_{n-k}a_{k+1} = 0$. But by the substitution $l = n-1-k$, we have



    $$ sum_{k=0}^{n-1} (-1)^k a_{n-k}a_{k+1}
    = - sum_{l=0}^{n-1} (-1)^l a_{l+1}a_{n-l}. $$



    (Here the parity of $n$ is used.) So the sum equals its negation, hence is zero as required.





    Solution 3 - using multivariate calculus. Let $mathcal{C}_r$ denote the curve defined by $x^r + y^r = 1$ in the first quadrant, oriented to the right. Then



    $$ I(r) := int_{0}^{1} left( (1 -x^r)^{1/r} - x right)^n , mathrm{d}x
    = int_{mathcal{C}_r} ( y - x )^n , mathrm{d}x. $$



    Notice that if $0 < r < s$, then $mathcal{C}_s$ lies above $mathcal{C}_r$, and so, the curve $mathcal{C}_r - mathcal{C}_s$ bounds some region, which we denote by $mathcal{D}$, counter-clockwise:



    $hspace{10em}$ Region D and two enclosing curves



    Then by Green's theorem,



    $$ I(r) - I(s)
    = int_{partial mathcal{D}} ( y - x )^n , mathrm{d}x
    = - iint_{mathcal{D}} n (y - x)^{n-1} , mathrm{d}xmathrm{d}y. $$



    But since the region $mathcal{D}$ is symmetric around $y = x$ and $n$ is even, interchanging the roles of $x$ and $y$ shows



    $$ iint_{mathcal{D}} n (y - x)^{n-1} , mathrm{d}xmathrm{d}y
    = iint_{mathcal{D}} n (x - y)^{n-1} , mathrm{d}xmathrm{d}y
    = - iint_{mathcal{D}} n (y - x)^{n-1} , mathrm{d}xmathrm{d}y. $$



    Therefore $I(r) = I(s)$ for any $ r < s$, and in particular, letting $s to infty$ gives



    $$ I(r) = int _{0}^{1} (1 - x)^n , mathrm{d}x = frac{1}{n+1}. $$






    share|cite|improve this answer











    $endgroup$









    • 2




      $begingroup$
      Beautiful! This looks like Glasser's Master theorem little brother :D
      $endgroup$
      – Zacky
      Jan 12 at 1:02












    • $begingroup$
      Was it necessary to start with absolute value in the argument of function?
      $endgroup$
      – user
      Jan 12 at 1:04










    • $begingroup$
      @user, It is kind of necessary, in the sense that $$ int_{0}^{1} gleft(x-(1-x^r)^{1/r}right) , mathrm{d}x = int_{0}^{1} g(u) , mathrm{d}u $$ may fail if $g$ is not an even function on $[-1, 1]$.
      $endgroup$
      – Sangchul Lee
      Jan 12 at 1:06








    • 1




      $begingroup$
      @Zacky, Both Glasser's master theorem and my answer deals with specific examples of measure-preserving transformations, hence the conclusion should look similar. Of course, the beauty of Glasser's result is that its proof is very elementary. (The result itself was known much prior to his paper.)
      $endgroup$
      – Sangchul Lee
      Jan 12 at 1:12








    • 2




      $begingroup$
      @ersh, Thank you, and don't feel ashamed :) I was just lucky enough to find these approaches. It is like seeing facebook postings, which are basically a collage of someone else's finest moments...
      $endgroup$
      – Sangchul Lee
      Mar 17 at 7:37














    35





    +50







    35





    +50



    35




    +50



    $begingroup$

    We present 3 different solutions.





    Solution 1 - slick substitution. We prove a more general statement:




    Proposition. Let $R in (0, infty]$ and let $varphi : [0, R] to [0, R]$ satisfy the following conditions:





    • $varphi$ is continuous on $[0, R]$;


    • $varphi(0) = R$ and $varphi(R) = 0$;


    • $varphi$ is bijective and $varphi^{-1} = varphi$.


    Then for any integrable function $f$ on $[0, R]$,
    $$ int_{0}^{R} f(|x-varphi(x)|) , mathrm{d}x = int_{0}^{R} f(x) , mathrm{d}x. $$




    Proof. In case $varphi$ is also continuously differentiable on $(0, R)$, by the substitution $y = varphi(x)$, or equivalently, $x = varphi(y)$,



    $$ I
    := int_{0}^{R} f( |x - varphi(x)| ) , mathrm{d}x
    = -int_{0}^{R} f( |varphi(y) - y| ) varphi'(y) , mathrm{d}y. $$



    Summing two integrals,



    begin{align*}
    2I
    &= int_{0}^{R} f( |x - varphi(x)| ) (1 - varphi'(x)) , mathrm{d}x \
    &= int_{-R}^{R} f( |u| ) , mathrm{d}u = 2int_{0}^{R} f(u) , mathrm{d}u, tag{$u = x - varphi(x)$}
    end{align*}



    proving the claim when $varphi$ is continuously differentiable. This proof can be easily adapted to general $varphi$ by using Stieltjes integral. ■



    Now plug $varphi(x) = (1-x^r)^{1/r}$ with $R = 1$ and $f(x) = x^n$ for positive even integer $n$. Then



    $$ int_{0}^{1} left( (1-x^r)^{1/r} - x right)^n , mathrm{d}x
    = int_{0}^{1} left| x - (1-x^r)^{1/r} right|^n , mathrm{d}x
    = int_{0}^{1} x^n , mathrm{d}x
    = frac{1}{n+1}. $$





    Solution 2 - using beta function. Here is an alternative solution. Write $p = 1/r$. Then using the substitution $x = u^p$,



    begin{align*}
    int_{0}^{1} left( (1 - x^r)^{1/r} - x right)^n , mathrm{d}x
    &= int_{0}^{1} left( (1 - u)^{p} - u^p right)^{n} pu^{p-1} , mathrm{d}u \
    &= sum_{k=0}^{n} (-1)^k binom{n}{k} p int_{0}^{1} (1-u)^{p(n-k)} u^{p(k+1)-1} , mathrm{d}u \
    &= sum_{k=0}^{n} (-1)^k binom{n}{k} p cdot frac{(p(n-k))!(p(k+1)-1)!}{(p(n+1))!}
    end{align*}



    Here, $s! = Gamma(s+1)$. Now define $a_k = (pk)!/k!$. Then the above sum simplifies to



    begin{align*}
    int_{0}^{1} left( (1 - x^r)^{1/r} - x right)^n , mathrm{d}x
    &= frac{1}{(n+1)a_{n+1}} sum_{k=0}^{n} (-1)^k a_{n-k}a_{k+1} \
    &= frac{1}{(n+1)a_{n+1}} left( a_0 a_{n+1} + sum_{k=0}^{n-1} (-1)^k a_{n-k}a_{k+1} right).
    end{align*}



    So it suffices to show that $sum_{k=0}^{n-1} (-1)^k a_{n-k}a_{k+1} = 0$. But by the substitution $l = n-1-k$, we have



    $$ sum_{k=0}^{n-1} (-1)^k a_{n-k}a_{k+1}
    = - sum_{l=0}^{n-1} (-1)^l a_{l+1}a_{n-l}. $$



    (Here the parity of $n$ is used.) So the sum equals its negation, hence is zero as required.





    Solution 3 - using multivariate calculus. Let $mathcal{C}_r$ denote the curve defined by $x^r + y^r = 1$ in the first quadrant, oriented to the right. Then



    $$ I(r) := int_{0}^{1} left( (1 -x^r)^{1/r} - x right)^n , mathrm{d}x
    = int_{mathcal{C}_r} ( y - x )^n , mathrm{d}x. $$



    Notice that if $0 < r < s$, then $mathcal{C}_s$ lies above $mathcal{C}_r$, and so, the curve $mathcal{C}_r - mathcal{C}_s$ bounds some region, which we denote by $mathcal{D}$, counter-clockwise:



    $hspace{10em}$ Region D and two enclosing curves



    Then by Green's theorem,



    $$ I(r) - I(s)
    = int_{partial mathcal{D}} ( y - x )^n , mathrm{d}x
    = - iint_{mathcal{D}} n (y - x)^{n-1} , mathrm{d}xmathrm{d}y. $$



    But since the region $mathcal{D}$ is symmetric around $y = x$ and $n$ is even, interchanging the roles of $x$ and $y$ shows



    $$ iint_{mathcal{D}} n (y - x)^{n-1} , mathrm{d}xmathrm{d}y
    = iint_{mathcal{D}} n (x - y)^{n-1} , mathrm{d}xmathrm{d}y
    = - iint_{mathcal{D}} n (y - x)^{n-1} , mathrm{d}xmathrm{d}y. $$



    Therefore $I(r) = I(s)$ for any $ r < s$, and in particular, letting $s to infty$ gives



    $$ I(r) = int _{0}^{1} (1 - x)^n , mathrm{d}x = frac{1}{n+1}. $$






    share|cite|improve this answer











    $endgroup$



    We present 3 different solutions.





    Solution 1 - slick substitution. We prove a more general statement:




    Proposition. Let $R in (0, infty]$ and let $varphi : [0, R] to [0, R]$ satisfy the following conditions:





    • $varphi$ is continuous on $[0, R]$;


    • $varphi(0) = R$ and $varphi(R) = 0$;


    • $varphi$ is bijective and $varphi^{-1} = varphi$.


    Then for any integrable function $f$ on $[0, R]$,
    $$ int_{0}^{R} f(|x-varphi(x)|) , mathrm{d}x = int_{0}^{R} f(x) , mathrm{d}x. $$




    Proof. In case $varphi$ is also continuously differentiable on $(0, R)$, by the substitution $y = varphi(x)$, or equivalently, $x = varphi(y)$,



    $$ I
    := int_{0}^{R} f( |x - varphi(x)| ) , mathrm{d}x
    = -int_{0}^{R} f( |varphi(y) - y| ) varphi'(y) , mathrm{d}y. $$



    Summing two integrals,



    begin{align*}
    2I
    &= int_{0}^{R} f( |x - varphi(x)| ) (1 - varphi'(x)) , mathrm{d}x \
    &= int_{-R}^{R} f( |u| ) , mathrm{d}u = 2int_{0}^{R} f(u) , mathrm{d}u, tag{$u = x - varphi(x)$}
    end{align*}



    proving the claim when $varphi$ is continuously differentiable. This proof can be easily adapted to general $varphi$ by using Stieltjes integral. ■



    Now plug $varphi(x) = (1-x^r)^{1/r}$ with $R = 1$ and $f(x) = x^n$ for positive even integer $n$. Then



    $$ int_{0}^{1} left( (1-x^r)^{1/r} - x right)^n , mathrm{d}x
    = int_{0}^{1} left| x - (1-x^r)^{1/r} right|^n , mathrm{d}x
    = int_{0}^{1} x^n , mathrm{d}x
    = frac{1}{n+1}. $$





    Solution 2 - using beta function. Here is an alternative solution. Write $p = 1/r$. Then using the substitution $x = u^p$,



    begin{align*}
    int_{0}^{1} left( (1 - x^r)^{1/r} - x right)^n , mathrm{d}x
    &= int_{0}^{1} left( (1 - u)^{p} - u^p right)^{n} pu^{p-1} , mathrm{d}u \
    &= sum_{k=0}^{n} (-1)^k binom{n}{k} p int_{0}^{1} (1-u)^{p(n-k)} u^{p(k+1)-1} , mathrm{d}u \
    &= sum_{k=0}^{n} (-1)^k binom{n}{k} p cdot frac{(p(n-k))!(p(k+1)-1)!}{(p(n+1))!}
    end{align*}



    Here, $s! = Gamma(s+1)$. Now define $a_k = (pk)!/k!$. Then the above sum simplifies to



    begin{align*}
    int_{0}^{1} left( (1 - x^r)^{1/r} - x right)^n , mathrm{d}x
    &= frac{1}{(n+1)a_{n+1}} sum_{k=0}^{n} (-1)^k a_{n-k}a_{k+1} \
    &= frac{1}{(n+1)a_{n+1}} left( a_0 a_{n+1} + sum_{k=0}^{n-1} (-1)^k a_{n-k}a_{k+1} right).
    end{align*}



    So it suffices to show that $sum_{k=0}^{n-1} (-1)^k a_{n-k}a_{k+1} = 0$. But by the substitution $l = n-1-k$, we have



    $$ sum_{k=0}^{n-1} (-1)^k a_{n-k}a_{k+1}
    = - sum_{l=0}^{n-1} (-1)^l a_{l+1}a_{n-l}. $$



    (Here the parity of $n$ is used.) So the sum equals its negation, hence is zero as required.





    Solution 3 - using multivariate calculus. Let $mathcal{C}_r$ denote the curve defined by $x^r + y^r = 1$ in the first quadrant, oriented to the right. Then



    $$ I(r) := int_{0}^{1} left( (1 -x^r)^{1/r} - x right)^n , mathrm{d}x
    = int_{mathcal{C}_r} ( y - x )^n , mathrm{d}x. $$



    Notice that if $0 < r < s$, then $mathcal{C}_s$ lies above $mathcal{C}_r$, and so, the curve $mathcal{C}_r - mathcal{C}_s$ bounds some region, which we denote by $mathcal{D}$, counter-clockwise:



    $hspace{10em}$ Region D and two enclosing curves



    Then by Green's theorem,



    $$ I(r) - I(s)
    = int_{partial mathcal{D}} ( y - x )^n , mathrm{d}x
    = - iint_{mathcal{D}} n (y - x)^{n-1} , mathrm{d}xmathrm{d}y. $$



    But since the region $mathcal{D}$ is symmetric around $y = x$ and $n$ is even, interchanging the roles of $x$ and $y$ shows



    $$ iint_{mathcal{D}} n (y - x)^{n-1} , mathrm{d}xmathrm{d}y
    = iint_{mathcal{D}} n (x - y)^{n-1} , mathrm{d}xmathrm{d}y
    = - iint_{mathcal{D}} n (y - x)^{n-1} , mathrm{d}xmathrm{d}y. $$



    Therefore $I(r) = I(s)$ for any $ r < s$, and in particular, letting $s to infty$ gives



    $$ I(r) = int _{0}^{1} (1 - x)^n , mathrm{d}x = frac{1}{n+1}. $$







    share|cite|improve this answer














    share|cite|improve this answer



    share|cite|improve this answer








    edited Mar 27 at 23:13









    Larry

    2,53031131




    2,53031131










    answered Jan 12 at 0:53









    Sangchul LeeSangchul Lee

    96.3k12171282




    96.3k12171282








    • 2




      $begingroup$
      Beautiful! This looks like Glasser's Master theorem little brother :D
      $endgroup$
      – Zacky
      Jan 12 at 1:02












    • $begingroup$
      Was it necessary to start with absolute value in the argument of function?
      $endgroup$
      – user
      Jan 12 at 1:04










    • $begingroup$
      @user, It is kind of necessary, in the sense that $$ int_{0}^{1} gleft(x-(1-x^r)^{1/r}right) , mathrm{d}x = int_{0}^{1} g(u) , mathrm{d}u $$ may fail if $g$ is not an even function on $[-1, 1]$.
      $endgroup$
      – Sangchul Lee
      Jan 12 at 1:06








    • 1




      $begingroup$
      @Zacky, Both Glasser's master theorem and my answer deals with specific examples of measure-preserving transformations, hence the conclusion should look similar. Of course, the beauty of Glasser's result is that its proof is very elementary. (The result itself was known much prior to his paper.)
      $endgroup$
      – Sangchul Lee
      Jan 12 at 1:12








    • 2




      $begingroup$
      @ersh, Thank you, and don't feel ashamed :) I was just lucky enough to find these approaches. It is like seeing facebook postings, which are basically a collage of someone else's finest moments...
      $endgroup$
      – Sangchul Lee
      Mar 17 at 7:37














    • 2




      $begingroup$
      Beautiful! This looks like Glasser's Master theorem little brother :D
      $endgroup$
      – Zacky
      Jan 12 at 1:02












    • $begingroup$
      Was it necessary to start with absolute value in the argument of function?
      $endgroup$
      – user
      Jan 12 at 1:04










    • $begingroup$
      @user, It is kind of necessary, in the sense that $$ int_{0}^{1} gleft(x-(1-x^r)^{1/r}right) , mathrm{d}x = int_{0}^{1} g(u) , mathrm{d}u $$ may fail if $g$ is not an even function on $[-1, 1]$.
      $endgroup$
      – Sangchul Lee
      Jan 12 at 1:06








    • 1




      $begingroup$
      @Zacky, Both Glasser's master theorem and my answer deals with specific examples of measure-preserving transformations, hence the conclusion should look similar. Of course, the beauty of Glasser's result is that its proof is very elementary. (The result itself was known much prior to his paper.)
      $endgroup$
      – Sangchul Lee
      Jan 12 at 1:12








    • 2




      $begingroup$
      @ersh, Thank you, and don't feel ashamed :) I was just lucky enough to find these approaches. It is like seeing facebook postings, which are basically a collage of someone else's finest moments...
      $endgroup$
      – Sangchul Lee
      Mar 17 at 7:37








    2




    2




    $begingroup$
    Beautiful! This looks like Glasser's Master theorem little brother :D
    $endgroup$
    – Zacky
    Jan 12 at 1:02






    $begingroup$
    Beautiful! This looks like Glasser's Master theorem little brother :D
    $endgroup$
    – Zacky
    Jan 12 at 1:02














    $begingroup$
    Was it necessary to start with absolute value in the argument of function?
    $endgroup$
    – user
    Jan 12 at 1:04




    $begingroup$
    Was it necessary to start with absolute value in the argument of function?
    $endgroup$
    – user
    Jan 12 at 1:04












    $begingroup$
    @user, It is kind of necessary, in the sense that $$ int_{0}^{1} gleft(x-(1-x^r)^{1/r}right) , mathrm{d}x = int_{0}^{1} g(u) , mathrm{d}u $$ may fail if $g$ is not an even function on $[-1, 1]$.
    $endgroup$
    – Sangchul Lee
    Jan 12 at 1:06






    $begingroup$
    @user, It is kind of necessary, in the sense that $$ int_{0}^{1} gleft(x-(1-x^r)^{1/r}right) , mathrm{d}x = int_{0}^{1} g(u) , mathrm{d}u $$ may fail if $g$ is not an even function on $[-1, 1]$.
    $endgroup$
    – Sangchul Lee
    Jan 12 at 1:06






    1




    1




    $begingroup$
    @Zacky, Both Glasser's master theorem and my answer deals with specific examples of measure-preserving transformations, hence the conclusion should look similar. Of course, the beauty of Glasser's result is that its proof is very elementary. (The result itself was known much prior to his paper.)
    $endgroup$
    – Sangchul Lee
    Jan 12 at 1:12






    $begingroup$
    @Zacky, Both Glasser's master theorem and my answer deals with specific examples of measure-preserving transformations, hence the conclusion should look similar. Of course, the beauty of Glasser's result is that its proof is very elementary. (The result itself was known much prior to his paper.)
    $endgroup$
    – Sangchul Lee
    Jan 12 at 1:12






    2




    2




    $begingroup$
    @ersh, Thank you, and don't feel ashamed :) I was just lucky enough to find these approaches. It is like seeing facebook postings, which are basically a collage of someone else's finest moments...
    $endgroup$
    – Sangchul Lee
    Mar 17 at 7:37




    $begingroup$
    @ersh, Thank you, and don't feel ashamed :) I was just lucky enough to find these approaches. It is like seeing facebook postings, which are basically a collage of someone else's finest moments...
    $endgroup$
    – Sangchul Lee
    Mar 17 at 7:37











    4












    $begingroup$

    Here's your Beta integral
    $$S=int_0^1x^{n-k}(1-x^r)^{k/r}dx$$
    Setting $w=x^r$, we see that
    $$S=frac1rint_0^1w^{frac{n+1-k-r}r}(1-w)^{k/r}dw$$
    $$S=frac1rint_0^1w^{frac{n+1-k}r-1}(1-w)^{frac{k+r}r-1}dw$$
    $$S=frac1rmathrm{B}bigg(frac{n+1-k}r,frac{k+r}rbigg)$$
    So
    $$I(r,n)=int_0^1[(1-x^r)^{1/r}-x]^ndx$$
    $$I(r,n)=frac1rsum_{k=0}^{n}(-1)^{n-k}{nchoose k}frac{Gamma(frac{n+1-k}r)Gamma(frac{k+r}r)}{Gamma(1+frac{n+1}r)}$$
    $$I(r,n)=frac1{r}frac{Gamma(n+1)}{Gamma(1+frac{n+1}r)}sum_{k=0}^{n}(-1)^{n-k}frac{Gamma(frac{n+1-k}r)Gamma(frac{k+r}r)}{Gamma(k+1)Gamma(n-k+1)}$$



    Which is a closed form






    share|cite|improve this answer











    $endgroup$









    • 1




      $begingroup$
      Try to find an error in your derivation as $I(r,n)=frac{1}{n+1}$ for even $n$ and any $r$.
      $endgroup$
      – user
      Jan 12 at 1:45












    • $begingroup$
      @user Ah yes I forgot $n$ was even
      $endgroup$
      – clathratus
      Jan 12 at 1:57






    • 1




      $begingroup$
      Nice approach. Seems almost obvious after reading it, but I was stuck when I first saw it (+1)
      $endgroup$
      – user150203
      Jan 12 at 5:31
















    4












    $begingroup$

    Here's your Beta integral
    $$S=int_0^1x^{n-k}(1-x^r)^{k/r}dx$$
    Setting $w=x^r$, we see that
    $$S=frac1rint_0^1w^{frac{n+1-k-r}r}(1-w)^{k/r}dw$$
    $$S=frac1rint_0^1w^{frac{n+1-k}r-1}(1-w)^{frac{k+r}r-1}dw$$
    $$S=frac1rmathrm{B}bigg(frac{n+1-k}r,frac{k+r}rbigg)$$
    So
    $$I(r,n)=int_0^1[(1-x^r)^{1/r}-x]^ndx$$
    $$I(r,n)=frac1rsum_{k=0}^{n}(-1)^{n-k}{nchoose k}frac{Gamma(frac{n+1-k}r)Gamma(frac{k+r}r)}{Gamma(1+frac{n+1}r)}$$
    $$I(r,n)=frac1{r}frac{Gamma(n+1)}{Gamma(1+frac{n+1}r)}sum_{k=0}^{n}(-1)^{n-k}frac{Gamma(frac{n+1-k}r)Gamma(frac{k+r}r)}{Gamma(k+1)Gamma(n-k+1)}$$



    Which is a closed form






    share|cite|improve this answer











    $endgroup$









    • 1




      $begingroup$
      Try to find an error in your derivation as $I(r,n)=frac{1}{n+1}$ for even $n$ and any $r$.
      $endgroup$
      – user
      Jan 12 at 1:45












    • $begingroup$
      @user Ah yes I forgot $n$ was even
      $endgroup$
      – clathratus
      Jan 12 at 1:57






    • 1




      $begingroup$
      Nice approach. Seems almost obvious after reading it, but I was stuck when I first saw it (+1)
      $endgroup$
      – user150203
      Jan 12 at 5:31














    4












    4








    4





    $begingroup$

    Here's your Beta integral
    $$S=int_0^1x^{n-k}(1-x^r)^{k/r}dx$$
    Setting $w=x^r$, we see that
    $$S=frac1rint_0^1w^{frac{n+1-k-r}r}(1-w)^{k/r}dw$$
    $$S=frac1rint_0^1w^{frac{n+1-k}r-1}(1-w)^{frac{k+r}r-1}dw$$
    $$S=frac1rmathrm{B}bigg(frac{n+1-k}r,frac{k+r}rbigg)$$
    So
    $$I(r,n)=int_0^1[(1-x^r)^{1/r}-x]^ndx$$
    $$I(r,n)=frac1rsum_{k=0}^{n}(-1)^{n-k}{nchoose k}frac{Gamma(frac{n+1-k}r)Gamma(frac{k+r}r)}{Gamma(1+frac{n+1}r)}$$
    $$I(r,n)=frac1{r}frac{Gamma(n+1)}{Gamma(1+frac{n+1}r)}sum_{k=0}^{n}(-1)^{n-k}frac{Gamma(frac{n+1-k}r)Gamma(frac{k+r}r)}{Gamma(k+1)Gamma(n-k+1)}$$



    Which is a closed form






    share|cite|improve this answer











    $endgroup$



    Here's your Beta integral
    $$S=int_0^1x^{n-k}(1-x^r)^{k/r}dx$$
    Setting $w=x^r$, we see that
    $$S=frac1rint_0^1w^{frac{n+1-k-r}r}(1-w)^{k/r}dw$$
    $$S=frac1rint_0^1w^{frac{n+1-k}r-1}(1-w)^{frac{k+r}r-1}dw$$
    $$S=frac1rmathrm{B}bigg(frac{n+1-k}r,frac{k+r}rbigg)$$
    So
    $$I(r,n)=int_0^1[(1-x^r)^{1/r}-x]^ndx$$
    $$I(r,n)=frac1rsum_{k=0}^{n}(-1)^{n-k}{nchoose k}frac{Gamma(frac{n+1-k}r)Gamma(frac{k+r}r)}{Gamma(1+frac{n+1}r)}$$
    $$I(r,n)=frac1{r}frac{Gamma(n+1)}{Gamma(1+frac{n+1}r)}sum_{k=0}^{n}(-1)^{n-k}frac{Gamma(frac{n+1-k}r)Gamma(frac{k+r}r)}{Gamma(k+1)Gamma(n-k+1)}$$



    Which is a closed form







    share|cite|improve this answer














    share|cite|improve this answer



    share|cite|improve this answer








    edited Jan 12 at 1:56

























    answered Jan 12 at 0:59









    clathratusclathratus

    5,0161438




    5,0161438








    • 1




      $begingroup$
      Try to find an error in your derivation as $I(r,n)=frac{1}{n+1}$ for even $n$ and any $r$.
      $endgroup$
      – user
      Jan 12 at 1:45












    • $begingroup$
      @user Ah yes I forgot $n$ was even
      $endgroup$
      – clathratus
      Jan 12 at 1:57






    • 1




      $begingroup$
      Nice approach. Seems almost obvious after reading it, but I was stuck when I first saw it (+1)
      $endgroup$
      – user150203
      Jan 12 at 5:31














    • 1




      $begingroup$
      Try to find an error in your derivation as $I(r,n)=frac{1}{n+1}$ for even $n$ and any $r$.
      $endgroup$
      – user
      Jan 12 at 1:45












    • $begingroup$
      @user Ah yes I forgot $n$ was even
      $endgroup$
      – clathratus
      Jan 12 at 1:57






    • 1




      $begingroup$
      Nice approach. Seems almost obvious after reading it, but I was stuck when I first saw it (+1)
      $endgroup$
      – user150203
      Jan 12 at 5:31








    1




    1




    $begingroup$
    Try to find an error in your derivation as $I(r,n)=frac{1}{n+1}$ for even $n$ and any $r$.
    $endgroup$
    – user
    Jan 12 at 1:45






    $begingroup$
    Try to find an error in your derivation as $I(r,n)=frac{1}{n+1}$ for even $n$ and any $r$.
    $endgroup$
    – user
    Jan 12 at 1:45














    $begingroup$
    @user Ah yes I forgot $n$ was even
    $endgroup$
    – clathratus
    Jan 12 at 1:57




    $begingroup$
    @user Ah yes I forgot $n$ was even
    $endgroup$
    – clathratus
    Jan 12 at 1:57




    1




    1




    $begingroup$
    Nice approach. Seems almost obvious after reading it, but I was stuck when I first saw it (+1)
    $endgroup$
    – user150203
    Jan 12 at 5:31




    $begingroup$
    Nice approach. Seems almost obvious after reading it, but I was stuck when I first saw it (+1)
    $endgroup$
    – user150203
    Jan 12 at 5:31











    4












    $begingroup$

    Here's a proof with Hypergeometirc function.



    We have
    $$
    underset{j=1}{overset{2 n+1}{sum }}
    left(
    begin{array}{c}
    2 n \
    j-1 \
    end{array}
    right)
    (-x)^{j-1}
    left(left(1-x^rright)^{1/r}right)^{-j+2 n+1}
    =left(left(1-x^rright)^{1/r}-xright)^{2 n}
    $$

    by binomial expansion.



    It is easy to verify that
    $$
    left(
    begin{array}{c}
    2 n \
    j-1 \
    end{array}
    right)
    (-x)^{j-1}
    left(left(1-x^rright)^{1/r}right)^{-j+2 n+1}
    =
    frac{mathrm d}{mathrm d x}left(
    frac{1}{2 n+1}
    (-1)^{j+1} x^j binom{2 n+1}{j} , _2F_1left(frac{j}{r},-frac{-j+2 n+1}{r};frac{j}{r}+1;x^rright)
    right)
    $$

    Therefore, we have
    $$
    int((1-x^r)^{1/r}-x)^{2 n} mathrm dx
    =
    sum _{j=1}^{2 n+1} frac{1}{2 n+1} (-1)^{j+1} x^j binom{2 n+1}{j} , _2F_1left(frac{j}{r},-frac{-j+2 n+1}{r};frac{j}{r}+1;x^rright).
    $$

    When $j=2n+1$, the summand in the right hand equals $frac{x^{2 n+1}}{2 n+1}$. This is the term which gives us $frac 1 {2n+1}$.






    share|cite|improve this answer









    $endgroup$


















      4












      $begingroup$

      Here's a proof with Hypergeometirc function.



      We have
      $$
      underset{j=1}{overset{2 n+1}{sum }}
      left(
      begin{array}{c}
      2 n \
      j-1 \
      end{array}
      right)
      (-x)^{j-1}
      left(left(1-x^rright)^{1/r}right)^{-j+2 n+1}
      =left(left(1-x^rright)^{1/r}-xright)^{2 n}
      $$

      by binomial expansion.



      It is easy to verify that
      $$
      left(
      begin{array}{c}
      2 n \
      j-1 \
      end{array}
      right)
      (-x)^{j-1}
      left(left(1-x^rright)^{1/r}right)^{-j+2 n+1}
      =
      frac{mathrm d}{mathrm d x}left(
      frac{1}{2 n+1}
      (-1)^{j+1} x^j binom{2 n+1}{j} , _2F_1left(frac{j}{r},-frac{-j+2 n+1}{r};frac{j}{r}+1;x^rright)
      right)
      $$

      Therefore, we have
      $$
      int((1-x^r)^{1/r}-x)^{2 n} mathrm dx
      =
      sum _{j=1}^{2 n+1} frac{1}{2 n+1} (-1)^{j+1} x^j binom{2 n+1}{j} , _2F_1left(frac{j}{r},-frac{-j+2 n+1}{r};frac{j}{r}+1;x^rright).
      $$

      When $j=2n+1$, the summand in the right hand equals $frac{x^{2 n+1}}{2 n+1}$. This is the term which gives us $frac 1 {2n+1}$.






      share|cite|improve this answer









      $endgroup$
















        4












        4








        4





        $begingroup$

        Here's a proof with Hypergeometirc function.



        We have
        $$
        underset{j=1}{overset{2 n+1}{sum }}
        left(
        begin{array}{c}
        2 n \
        j-1 \
        end{array}
        right)
        (-x)^{j-1}
        left(left(1-x^rright)^{1/r}right)^{-j+2 n+1}
        =left(left(1-x^rright)^{1/r}-xright)^{2 n}
        $$

        by binomial expansion.



        It is easy to verify that
        $$
        left(
        begin{array}{c}
        2 n \
        j-1 \
        end{array}
        right)
        (-x)^{j-1}
        left(left(1-x^rright)^{1/r}right)^{-j+2 n+1}
        =
        frac{mathrm d}{mathrm d x}left(
        frac{1}{2 n+1}
        (-1)^{j+1} x^j binom{2 n+1}{j} , _2F_1left(frac{j}{r},-frac{-j+2 n+1}{r};frac{j}{r}+1;x^rright)
        right)
        $$

        Therefore, we have
        $$
        int((1-x^r)^{1/r}-x)^{2 n} mathrm dx
        =
        sum _{j=1}^{2 n+1} frac{1}{2 n+1} (-1)^{j+1} x^j binom{2 n+1}{j} , _2F_1left(frac{j}{r},-frac{-j+2 n+1}{r};frac{j}{r}+1;x^rright).
        $$

        When $j=2n+1$, the summand in the right hand equals $frac{x^{2 n+1}}{2 n+1}$. This is the term which gives us $frac 1 {2n+1}$.






        share|cite|improve this answer









        $endgroup$



        Here's a proof with Hypergeometirc function.



        We have
        $$
        underset{j=1}{overset{2 n+1}{sum }}
        left(
        begin{array}{c}
        2 n \
        j-1 \
        end{array}
        right)
        (-x)^{j-1}
        left(left(1-x^rright)^{1/r}right)^{-j+2 n+1}
        =left(left(1-x^rright)^{1/r}-xright)^{2 n}
        $$

        by binomial expansion.



        It is easy to verify that
        $$
        left(
        begin{array}{c}
        2 n \
        j-1 \
        end{array}
        right)
        (-x)^{j-1}
        left(left(1-x^rright)^{1/r}right)^{-j+2 n+1}
        =
        frac{mathrm d}{mathrm d x}left(
        frac{1}{2 n+1}
        (-1)^{j+1} x^j binom{2 n+1}{j} , _2F_1left(frac{j}{r},-frac{-j+2 n+1}{r};frac{j}{r}+1;x^rright)
        right)
        $$

        Therefore, we have
        $$
        int((1-x^r)^{1/r}-x)^{2 n} mathrm dx
        =
        sum _{j=1}^{2 n+1} frac{1}{2 n+1} (-1)^{j+1} x^j binom{2 n+1}{j} , _2F_1left(frac{j}{r},-frac{-j+2 n+1}{r};frac{j}{r}+1;x^rright).
        $$

        When $j=2n+1$, the summand in the right hand equals $frac{x^{2 n+1}}{2 n+1}$. This is the term which gives us $frac 1 {2n+1}$.







        share|cite|improve this answer












        share|cite|improve this answer



        share|cite|improve this answer










        answered Jan 12 at 11:03









        ablmfablmf

        2,58052452




        2,58052452























            3












            $begingroup$

            $newcommand{bbx}[1]{,bbox[15px,border:1px groove navy]{displaystyle{#1}},}
            newcommand{braces}[1]{leftlbrace,{#1},rightrbrace}
            newcommand{bracks}[1]{leftlbrack,{#1},rightrbrack}
            newcommand{dd}{mathrm{d}}
            newcommand{ds}[1]{displaystyle{#1}}
            newcommand{expo}[1]{,mathrm{e}^{#1},}
            newcommand{ic}{mathrm{i}}
            newcommand{mc}[1]{mathcal{#1}}
            newcommand{mrm}[1]{mathrm{#1}}
            newcommand{pars}[1]{left(,{#1},right)}
            newcommand{partiald}[3][]{frac{partial^{#1} #2}{partial #3^{#1}}}
            newcommand{root}[2][]{,sqrt[#1]{,{#2},},}
            newcommand{totald}[3][]{frac{mathrm{d}^{#1} #2}{mathrm{d} #3^{#1}}}
            newcommand{verts}[1]{leftvert,{#1},rightvert}$

            begin{align}
            &bbox[10px,#ffd]{left.int_{0}^{1}bracks{pars{1 - x^{r}}^{1/r} - x}^{2k},dd x,rightvert_{{large r > 0} atop
            {large k in mathbb{N}_{geq 0}}}}
            \[5mm] stackrel{x^{large r} mapsto x}{=},,,&
            int_{0}^{1}bracks{pars{1 - x}^{1/r} - x^{1/r}}^{2k},{1 over r},
            x^{1/r - 1},dd x
            \[5mm]
            ,,,stackrel{x mapsto x + 1/2}{=},,,&
            {1 over r}int_{-1/2}^{1/2}bracks{pars{{1 over 2} - x}^{1/r} - pars{{1 over 2} + x}^{1/r}}^{2k},
            pars{{1 over 2} + x}^{1/r - 1},dd x
            \[8mm] = &
            {1 over r}int_{0}^{1/2}bracks{pars{{1 over 2} - x}^{1/r} - pars{{1 over 2} + x!}^{1/r}}^{2k}times
            \[2mm] &
            phantom{{1 over r}int_{0}^{1/2}}bracks{pars{{1 over 2} + x}^{1/r - 1} +
            pars{{1 over 2} - x}^{1/r - 1}!}!dd x
            \[8mm] = &
            -int_{0}^{1/2}{1 over 2k + 1},partiald{}{x}bracks{pars{{1 over 2} - x}^{1/r} - pars{{1 over 2} + x}^{1/r}}^{2k + 1},dd x
            \[5mm] = &
            underbrace{braces{-bracks{pars{{1 over 2} - x}^{1/r} - pars{{1 over 2} + x}^{1/r}}^{2k + 1}}_{x = 0}^{x = 1/2}}
            _{ds{= 1 - 0 = 1}},,,{1 over 2k + 1}
            \[5mm] = &
            bbx{1 over 2k + 1}
            end{align}






            share|cite|improve this answer











            $endgroup$













            • $begingroup$
              Nice solution, (+1).
              $endgroup$
              – Larry
              Mar 5 at 13:19










            • $begingroup$
              Thanks @Larry .
              $endgroup$
              – Felix Marin
              Mar 5 at 15:23
















            3












            $begingroup$

            $newcommand{bbx}[1]{,bbox[15px,border:1px groove navy]{displaystyle{#1}},}
            newcommand{braces}[1]{leftlbrace,{#1},rightrbrace}
            newcommand{bracks}[1]{leftlbrack,{#1},rightrbrack}
            newcommand{dd}{mathrm{d}}
            newcommand{ds}[1]{displaystyle{#1}}
            newcommand{expo}[1]{,mathrm{e}^{#1},}
            newcommand{ic}{mathrm{i}}
            newcommand{mc}[1]{mathcal{#1}}
            newcommand{mrm}[1]{mathrm{#1}}
            newcommand{pars}[1]{left(,{#1},right)}
            newcommand{partiald}[3][]{frac{partial^{#1} #2}{partial #3^{#1}}}
            newcommand{root}[2][]{,sqrt[#1]{,{#2},},}
            newcommand{totald}[3][]{frac{mathrm{d}^{#1} #2}{mathrm{d} #3^{#1}}}
            newcommand{verts}[1]{leftvert,{#1},rightvert}$

            begin{align}
            &bbox[10px,#ffd]{left.int_{0}^{1}bracks{pars{1 - x^{r}}^{1/r} - x}^{2k},dd x,rightvert_{{large r > 0} atop
            {large k in mathbb{N}_{geq 0}}}}
            \[5mm] stackrel{x^{large r} mapsto x}{=},,,&
            int_{0}^{1}bracks{pars{1 - x}^{1/r} - x^{1/r}}^{2k},{1 over r},
            x^{1/r - 1},dd x
            \[5mm]
            ,,,stackrel{x mapsto x + 1/2}{=},,,&
            {1 over r}int_{-1/2}^{1/2}bracks{pars{{1 over 2} - x}^{1/r} - pars{{1 over 2} + x}^{1/r}}^{2k},
            pars{{1 over 2} + x}^{1/r - 1},dd x
            \[8mm] = &
            {1 over r}int_{0}^{1/2}bracks{pars{{1 over 2} - x}^{1/r} - pars{{1 over 2} + x!}^{1/r}}^{2k}times
            \[2mm] &
            phantom{{1 over r}int_{0}^{1/2}}bracks{pars{{1 over 2} + x}^{1/r - 1} +
            pars{{1 over 2} - x}^{1/r - 1}!}!dd x
            \[8mm] = &
            -int_{0}^{1/2}{1 over 2k + 1},partiald{}{x}bracks{pars{{1 over 2} - x}^{1/r} - pars{{1 over 2} + x}^{1/r}}^{2k + 1},dd x
            \[5mm] = &
            underbrace{braces{-bracks{pars{{1 over 2} - x}^{1/r} - pars{{1 over 2} + x}^{1/r}}^{2k + 1}}_{x = 0}^{x = 1/2}}
            _{ds{= 1 - 0 = 1}},,,{1 over 2k + 1}
            \[5mm] = &
            bbx{1 over 2k + 1}
            end{align}






            share|cite|improve this answer











            $endgroup$













            • $begingroup$
              Nice solution, (+1).
              $endgroup$
              – Larry
              Mar 5 at 13:19










            • $begingroup$
              Thanks @Larry .
              $endgroup$
              – Felix Marin
              Mar 5 at 15:23














            3












            3








            3





            $begingroup$

            $newcommand{bbx}[1]{,bbox[15px,border:1px groove navy]{displaystyle{#1}},}
            newcommand{braces}[1]{leftlbrace,{#1},rightrbrace}
            newcommand{bracks}[1]{leftlbrack,{#1},rightrbrack}
            newcommand{dd}{mathrm{d}}
            newcommand{ds}[1]{displaystyle{#1}}
            newcommand{expo}[1]{,mathrm{e}^{#1},}
            newcommand{ic}{mathrm{i}}
            newcommand{mc}[1]{mathcal{#1}}
            newcommand{mrm}[1]{mathrm{#1}}
            newcommand{pars}[1]{left(,{#1},right)}
            newcommand{partiald}[3][]{frac{partial^{#1} #2}{partial #3^{#1}}}
            newcommand{root}[2][]{,sqrt[#1]{,{#2},},}
            newcommand{totald}[3][]{frac{mathrm{d}^{#1} #2}{mathrm{d} #3^{#1}}}
            newcommand{verts}[1]{leftvert,{#1},rightvert}$

            begin{align}
            &bbox[10px,#ffd]{left.int_{0}^{1}bracks{pars{1 - x^{r}}^{1/r} - x}^{2k},dd x,rightvert_{{large r > 0} atop
            {large k in mathbb{N}_{geq 0}}}}
            \[5mm] stackrel{x^{large r} mapsto x}{=},,,&
            int_{0}^{1}bracks{pars{1 - x}^{1/r} - x^{1/r}}^{2k},{1 over r},
            x^{1/r - 1},dd x
            \[5mm]
            ,,,stackrel{x mapsto x + 1/2}{=},,,&
            {1 over r}int_{-1/2}^{1/2}bracks{pars{{1 over 2} - x}^{1/r} - pars{{1 over 2} + x}^{1/r}}^{2k},
            pars{{1 over 2} + x}^{1/r - 1},dd x
            \[8mm] = &
            {1 over r}int_{0}^{1/2}bracks{pars{{1 over 2} - x}^{1/r} - pars{{1 over 2} + x!}^{1/r}}^{2k}times
            \[2mm] &
            phantom{{1 over r}int_{0}^{1/2}}bracks{pars{{1 over 2} + x}^{1/r - 1} +
            pars{{1 over 2} - x}^{1/r - 1}!}!dd x
            \[8mm] = &
            -int_{0}^{1/2}{1 over 2k + 1},partiald{}{x}bracks{pars{{1 over 2} - x}^{1/r} - pars{{1 over 2} + x}^{1/r}}^{2k + 1},dd x
            \[5mm] = &
            underbrace{braces{-bracks{pars{{1 over 2} - x}^{1/r} - pars{{1 over 2} + x}^{1/r}}^{2k + 1}}_{x = 0}^{x = 1/2}}
            _{ds{= 1 - 0 = 1}},,,{1 over 2k + 1}
            \[5mm] = &
            bbx{1 over 2k + 1}
            end{align}






            share|cite|improve this answer











            $endgroup$



            $newcommand{bbx}[1]{,bbox[15px,border:1px groove navy]{displaystyle{#1}},}
            newcommand{braces}[1]{leftlbrace,{#1},rightrbrace}
            newcommand{bracks}[1]{leftlbrack,{#1},rightrbrack}
            newcommand{dd}{mathrm{d}}
            newcommand{ds}[1]{displaystyle{#1}}
            newcommand{expo}[1]{,mathrm{e}^{#1},}
            newcommand{ic}{mathrm{i}}
            newcommand{mc}[1]{mathcal{#1}}
            newcommand{mrm}[1]{mathrm{#1}}
            newcommand{pars}[1]{left(,{#1},right)}
            newcommand{partiald}[3][]{frac{partial^{#1} #2}{partial #3^{#1}}}
            newcommand{root}[2][]{,sqrt[#1]{,{#2},},}
            newcommand{totald}[3][]{frac{mathrm{d}^{#1} #2}{mathrm{d} #3^{#1}}}
            newcommand{verts}[1]{leftvert,{#1},rightvert}$

            begin{align}
            &bbox[10px,#ffd]{left.int_{0}^{1}bracks{pars{1 - x^{r}}^{1/r} - x}^{2k},dd x,rightvert_{{large r > 0} atop
            {large k in mathbb{N}_{geq 0}}}}
            \[5mm] stackrel{x^{large r} mapsto x}{=},,,&
            int_{0}^{1}bracks{pars{1 - x}^{1/r} - x^{1/r}}^{2k},{1 over r},
            x^{1/r - 1},dd x
            \[5mm]
            ,,,stackrel{x mapsto x + 1/2}{=},,,&
            {1 over r}int_{-1/2}^{1/2}bracks{pars{{1 over 2} - x}^{1/r} - pars{{1 over 2} + x}^{1/r}}^{2k},
            pars{{1 over 2} + x}^{1/r - 1},dd x
            \[8mm] = &
            {1 over r}int_{0}^{1/2}bracks{pars{{1 over 2} - x}^{1/r} - pars{{1 over 2} + x!}^{1/r}}^{2k}times
            \[2mm] &
            phantom{{1 over r}int_{0}^{1/2}}bracks{pars{{1 over 2} + x}^{1/r - 1} +
            pars{{1 over 2} - x}^{1/r - 1}!}!dd x
            \[8mm] = &
            -int_{0}^{1/2}{1 over 2k + 1},partiald{}{x}bracks{pars{{1 over 2} - x}^{1/r} - pars{{1 over 2} + x}^{1/r}}^{2k + 1},dd x
            \[5mm] = &
            underbrace{braces{-bracks{pars{{1 over 2} - x}^{1/r} - pars{{1 over 2} + x}^{1/r}}^{2k + 1}}_{x = 0}^{x = 1/2}}
            _{ds{= 1 - 0 = 1}},,,{1 over 2k + 1}
            \[5mm] = &
            bbx{1 over 2k + 1}
            end{align}







            share|cite|improve this answer














            share|cite|improve this answer



            share|cite|improve this answer








            edited Mar 17 at 4:47

























            answered Mar 5 at 1:10









            Felix MarinFelix Marin

            68.9k7109146




            68.9k7109146












            • $begingroup$
              Nice solution, (+1).
              $endgroup$
              – Larry
              Mar 5 at 13:19










            • $begingroup$
              Thanks @Larry .
              $endgroup$
              – Felix Marin
              Mar 5 at 15:23


















            • $begingroup$
              Nice solution, (+1).
              $endgroup$
              – Larry
              Mar 5 at 13:19










            • $begingroup$
              Thanks @Larry .
              $endgroup$
              – Felix Marin
              Mar 5 at 15:23
















            $begingroup$
            Nice solution, (+1).
            $endgroup$
            – Larry
            Mar 5 at 13:19




            $begingroup$
            Nice solution, (+1).
            $endgroup$
            – Larry
            Mar 5 at 13:19












            $begingroup$
            Thanks @Larry .
            $endgroup$
            – Felix Marin
            Mar 5 at 15:23




            $begingroup$
            Thanks @Larry .
            $endgroup$
            – Felix Marin
            Mar 5 at 15:23


















            draft saved

            draft discarded




















































            Thanks for contributing an answer to Mathematics Stack Exchange!


            • Please be sure to answer the question. Provide details and share your research!

            But avoid



            • Asking for help, clarification, or responding to other answers.

            • Making statements based on opinion; back them up with references or personal experience.


            Use MathJax to format equations. MathJax reference.


            To learn more, see our tips on writing great answers.




            draft saved


            draft discarded














            StackExchange.ready(
            function () {
            StackExchange.openid.initPostLogin('.new-post-login', 'https%3a%2f%2fmath.stackexchange.com%2fquestions%2f3070440%2fways-to-show-that-int-011-xr1-r-xndx-frac1n1%23new-answer', 'question_page');
            }
            );

            Post as a guest















            Required, but never shown





















































            Required, but never shown














            Required, but never shown












            Required, but never shown







            Required, but never shown

































            Required, but never shown














            Required, but never shown












            Required, but never shown







            Required, but never shown







            Popular posts from this blog

            Magento 2 - Add success message with knockout Planned maintenance scheduled April 23, 2019 at 23:30 UTC (7:30pm US/Eastern) Announcing the arrival of Valued Associate #679: Cesar Manara Unicorn Meta Zoo #1: Why another podcast?Success / Error message on ajax request$.widget is not a function when loading a homepage after add custom jQuery on custom themeHow can bind jQuery to current document in Magento 2 When template load by ajaxRedirect page using plugin in Magento 2Magento 2 - Update quantity and totals of cart page without page reload?Magento 2: Quote data not loaded on knockout checkoutMagento 2 : I need to change add to cart success message after adding product into cart through pluginMagento 2.2.5 How to add additional products to cart from new checkout step?Magento 2 Add error/success message with knockoutCan't validate Post Code on checkout page

            Fil:Tokke komm.svg

            Where did Arya get these scars? Unicorn Meta Zoo #1: Why another podcast? Announcing the arrival of Valued Associate #679: Cesar Manara Favourite questions and answers from the 1st quarter of 2019Why did Arya refuse to end it?Has the pronunciation of Arya Stark's name changed?Has Arya forgiven people?Why did Arya Stark lose her vision?Why can Arya still use the faces?Has the Narrow Sea become narrower?Does Arya Stark know how to make poisons outside of the House of Black and White?Why did Nymeria leave Arya?Why did Arya not kill the Lannister soldiers she encountered in the Riverlands?What is the current canonical age of Sansa, Bran and Arya Stark?